スピントロニクス理論の基礎/9-1A

(4442d) 更新


[前の章へ] <<<<  スピントロニクス理論の基礎(目次) >>>> [次の章へ]

9-1 スカラー場により誘起される電荷密度 (1)

10/24 追記

以下では低温近似の下で電荷密度を求める。

ここで「低温近似」が出る意味について植田先生からコメントがあった。

ここでの計算には不純物散乱のみを取り入れており、フォノン散乱は見ていない。 高温では不純物散乱よりもフォノン散乱が効いてくるため、 議論の前提が低温でしか成り立たない。

フォノン散乱を無視している近似と、 以下の計算で \omega \hbar \sim 0 と見なした近似と、 どちらが強い低温近似であるか、後で知りたいところ。

追記ここまで

追記2:

佐野先生からのコメントで、 \omega \hbar \sim 0 とするときの「低温」はフェルミエネルギーに比べて温度が低いと言うことであり、 金属のフェルミエネルギーが 10^4K などと非常に高い温度に対応することを考えれば、 後者の意味での低温近似は室温付近でも成り立つことを教えていただいた。

これと比較すると前者については 室温よりずっと下から電気伝導度にフォノンが効いてくるから、 後者の低温近似の方がずっと条件の緩い近似であることがわかる。

不純物散乱にスカラー場のポテンシャルを加え電荷密度を求める

不純物散乱に加えてスカラー場 \phi によるポテンシャルを考える。

H=H_i+H_\phi\equiv H_0+V_i+V_\phi

H_i は自由電子が不純物散乱だけを受ける場合のハミルトニアン。

スカラー場によるポテンシャル項は次のように表される。

(9.1)

V_\phi&=e\int d^3r\phi(\bm r,t)c^\dagger(\bm r,t)c(\bm r,t)\\ &= e\int d^3r \Big[\sum_{\bm q}\int\frac{d\Omega}{2\pi}e^{-i\bm q\cdot\bm r}e^{i\Omega t}\phi_{\bm q,\Omega}\Big] \Big[\sum_{\bm k}\int\frac{d\omega}{2\pi}e^{-i\bm k\cdot\bm r}e^{i\omega t}c^\dagger_{\bm k,\omega}\Big] \Big[\sum_{\bm k'}\int\frac{d\omega'}{2\pi}e^{i\bm k'\cdot\bm r}e^{-i\omega' t}c_{\bm k',\omega'}\Big]\\ &= e\sum_{\bm q,\bm k,\bm k'}\int\frac{d\Omega}{2\pi}\int\frac{d\omega}{2\pi}\int\frac{d\omega'}{2\pi} e^{i(-\bm q-\bm k+\bm k')\cdot\bm r}e^{i(\Omega+\omega-\omega') t} \phi_{\bm q,\Omega}c^\dagger_{\bm k,\omega}c_{\bm k',\omega'} \\ &\stackrel{\red{?}}{=} e\sum_{\bm q,\bm k,\bm k'}\int\frac{d\Omega}{2\pi}\int\frac{d\omega}{2\pi}\int\frac{d\omega'}{2\pi} \phi_{\bm q,\Omega}c^\dagger_{\bm k,\omega}c_{\bm k',\omega'} \delta_{-\bm q-\bm k+\bm k',\bm0}\delta(\Omega+\omega-\omega') \\ &=e\sum_{\bm k,\bm q}\int\frac{d\omega}{2\phi}\int\frac{d\Omega}{2\phi}\phi(\bm q,\Omega)c_{\bm k,\omega}^\dagger c_{\bm k+\bm q,\omega+\Omega}

ただし、

(9.2)

\phi_{\bm q,\Omega}=\frac{\hbar}{V}\int d^3r \int dt e^{i\bm q\cdot\bm r}e^{-i\Omega t}\phi(\bm r,t)

\phi(\bm r,t)=\sum_{\bm q}\int\frac{d\Omega}{2\pi}e^{-i\bm q\cdot\bm r}e^{i\Omega t}\phi_{\bm q,\Omega}

これを元に、電荷密度を求めたいのだが、電荷密度は lesser Green 関数で表される。

(9.3)

\rho(\bm r,t)=\,\stackrel{\textcolor{red}{?}}{+}e\llangle \hat n_H\rrangle = e\llangle c_H^\dagger c_H\rrangle = -ie\Bigg[\ i\llangle c_H^\dagger c_H\rrangle\Bigg] = - ie{\red\hbar}G^<(\bm r,t,\bm r,t)

電子は負の電荷を持つので、符号は逆なのではないかと思ったのだけれど、 (9.69) でようやく説明があって、ここでは e<0 なのだそうだ。

\bm r=\bm r', t=t' と置いて、フーリエ係数の関係を導くと、

(9.3A)

&G^<(\bm r,t,\bm r,t)=\frac{1}{V} \int \frac{d\omega_1}{2\pi} \int \frac{d\omega_2}{2\pi} \sum_{\bm k_1,\bm k_2} e^{i(\bm k_1-\bm k_2)\cdot \bm r} e^{i(-\omega_1+\omega_2)t} G_{\bm k_1,\bm k_2,\omega_1,\omega_2}^<

\bm k_1=\bm k-\frac{\bm q}{2} \bm k_2=\bm k+\frac{\bm q}{2}

\omega_1=\omega-\frac{\Omega}{2} \omega_2=\omega+\frac{\Omega}{2}

と置けば、

(9.3B)

&\rho(\bm r,t)=-ie{\red\hbar}G^<(\bm r,t,\bm r,t)=-ie\frac{\textcolor{red}{\hbar}}{V} \int \frac{d\omega}{2\pi} \int \frac{d\Omega}{2\pi} \sum_{\bm k,\bm q} \textcolor{red}{e^{-i\bm q\cdot \bm r} e^{i\Omega t}} G_{\bm k-\frac{\bm q}{2},\bm k+\frac{\bm q}{2},\omega-\frac{\Omega}{2},\omega+\frac{\Omega}{2}}^<

ちょっとごまかしが

11/21 のセミナーで佐野先生にご指摘いただいたのは、 上記 (9.1) の4つ目の等号は、実は無条件で成り立つ物ではないこと。

e^{i(\Omega+\omega-\omega') t} を δ関数に直すためには時間での積分が必要であるが、 その積分はどこから来たのか?

実は、この段階では時間での積分は現われない。

したがってここでは、

(9.1')

V_\phi &= e\sum_{\bm q,\bm k,\bm k'}\int\frac{d\Omega}{2\pi}\int\frac{d\omega}{2\pi} e^{i(-\bm q-\bm k+\bm k')\cdot\bm r} \red{\left[\int\frac{d\omega'}{2\pi}e^{i(\Omega+\omega-\omega') t} \phi_{\bm q,\Omega}\right]}c^\dagger_{\bm k,\omega}c_{\bm k',\omega'}

までで変形をやめなければならない。

(9.4I) のフーリエ成分を求める際には うまくδ関数に化けて教科書通り進めばよいのだけれど、まだ確認できていない。

電場のあるときの lesser Green 関数 G< を不純物散乱の Green 関数 g で表す

G^< の Fourier 係数を求めるために (8.96) と同様に時間発展を自由+不純物散乱の部分と、 スカラー場による部分とに分けて書き、 深く考えずに同様の変形をしていく。

H=H_0+V_i+V_\phi

(8.96')

U=U_\phi U_i

ここで、 U_i H_i のみの時の解。

U=U_i U_{V_\phi} U_{V_\phi}=U_i^\dagger U

U_i=U_0 U_{V_i} U_{V_i}=U_0^\dagger U_i

U_{V_\phi}=U_{V_i}^\dagger U_0^\dagger U

(8.99')

& i\hbar \frac{\PD U_{V_\phi}}{\PD t} \\&= i\hbar \frac{\PD U_{V_i}^\dagger}{\PD t} U_0^\dagger U + i\hbar U_{V_i}^\dagger \frac{\PD U_0^\dagger}{\PD t} U + i\hbar U_{V_i}^\dagger U_0^\dagger \frac{\PD U}{\PD t} \\&= i\hbar \left( \frac{V_{iH_0} U_{V_i}}{i\hbar} \right)^\dagger U_0^\dagger U + i\hbar U_{V_i}^\dagger \left( \frac{H_0U_0}{i\hbar} \right)^\dagger U + i\hbar U_{V_i}^\dagger U_0^\dagger \left( \frac{HU}{i\hbar} \right) \\&= -U_{V_i}^\dagger V_{iH_0}^\dagger U_0^\dagger U -U_{V_i}^\dagger U_0^\dagger H_0^\dagger U +U_{V_i}^\dagger U_0^\dagger H U \\&= -U_{V_i}^\dagger (U_0^\dagger V_i U_0) U_0^\dagger U +U_{V_i}^\dagger U_0^\dagger (H-H_0) U \\&= -U_{V_i}^\dagger U_0^\dagger V_i U +U_{V_i}^\dagger U_0^\dagger (V_i+V_\phi) U \\&= U_{V_i}^\dagger U_0^\dagger V_\phi U \\&= (U_0 U_{V_i})^\dagger V_\phi (U_i U_i^\dagger) U \\&= U_i^\dagger V_\phi U_i (U_i^\dagger U) \\&= V_{\phi H_i} U_{V_\phi}

これを用いれば (8.101) と同様に、

(8.101')

G(\bm r,\tau,\bm r',\tau') \, \textcolor{red}{\stackrel{?}{=}} \, \Big\langle T_Ce^{-\frac{i}{\hbar}\int_C d\tau''V_{H_i}(\tau'')} c_{H_i}^\dagger(\bm r',\tau') c_{H_i}(\bm r,\tau) \Big\rangle

ただし (8.101) で指摘されたとおり、この書き換えにより

(9.4C)

U_{C_\beta}=e^{-\beta H(t_0)}

ではなく

(9.4D)

U_{C_\beta}=e^{-\beta V_\phi(t_0)}

になってしまっていることに注意が必要。

繰り返しになるが、これは運動エネルギーの項が無視されていることになり、 簡単には見過ごせない差になることが心配される。

(8.105) と同様に、

(9.4E)

&G(\bm r,\tau,\bm r',\tau')=g(\bm r,\tau,\bm r',\tau')\\ &+ \int_Cd\tau_1\int d^3r_1g(\bm r,\tau,\bm r_1,\tau_1)\times \frac{i}{\hbar}\Big\langle T_C\,e^{-\frac{i}{\hbar}\int_Cd\tau''H(\tau'')} [H_\phi(\tau_1),c(\bm r_1,\tau_1)]c^\dagger(\bm r',\tau')\Big\rangle\\

(8.108) と同様に、

(9.4F)

[H_\phi(\tau_1),c(\bm r_1,\tau_1)]=-e\phi(\bm r_1,\tau_1) c(\bm r_1,\tau_1)

となるから、

(8.109) と同様に、

(9.4G)

&G(\bm r,\tau,\bm r',\tau')=g(\bm r,\tau,\bm r',\tau')\\ &+ \int_Cd\tau_1\int d^3r_1g(\bm r,\tau,\bm r_1,\tau_1)\times \frac{1}{\hbar}\Big\langle T_C\,e^{-\frac{i}{\hbar}\int_Cd\tau''H(\tau'')} e\phi(\bm r_1,\tau_1)c(\bm r_1,\tau_1)c^\dagger(\bm r',\tau')\Big\rangle\\ &=g(\bm r,\tau,\bm r',\tau') +i\frac{e}{\hbar}\int_Cd\tau_1\int d^3r_1g(\bm r,\tau,\bm r_1,\tau_1) \phi(\bm r_1,\tau_1) G(\bm r_1,\tau_1,\bm r',\tau')

(8.111) と同様に、

(9.4H)

&G^<(\bm r,t,\bm r',t')= g^<(\bm r,t,\bm r',t')\\ &+\frac{e}{\hbar}\int_Cdt_1\int d^3r_1\left[ g^r(\bm r,t,\bm r_1,t_1) \phi(\bm r_1,t_1) G^<(\bm r_1,t_1,\bm r',t') +g^<(\bm r,t,\bm r_1,t_1) \phi(\bm r_1,t_1) G^a(\bm r_1,t_1,\bm r',t') \right]

(8.117) と同様に( \alpha=a,r )、ただし \omega が変化する余地を残して、

(9.4I)

&G^\alpha_{\bm k_1,\bm k_2,\omega_1,\omega_2}= 2\pi\delta(\omega_1-\omega_2)\delta_{\bm k_1,\bm k_2}g^<_{\bm k_1,\omega_1}\\ &+e\sum_{\bm q}\int d\frac{\Omega}{2\pi}\ g^\alpha_{\bm k_1,\omega_1} \phi(\bm q,\Omega) G^\alpha_{\bm k_1+\bm q,\bm k_2,\omega_1+\Omega,\omega_2}

(8.145) と同様に、ただし \omega が変化する余地を残して、

(9.4J)

&G^<_{\bm k_1,\bm k_2,\omega_1,\omega_2}= 2\pi\delta(\omega_1-\omega_2)\delta_{\bm k_1,\bm k_2}g^<_{\bm k_1,\omega_1}\\ &+e\sum_{\bm q}\int d\frac{\Omega}{2\pi}\left[ g^r_{\bm k_1,\omega_1} \phi(\bm q,\Omega) G^<_{\bm k_1+\bm q,\bm k_2,\omega_1+\Omega,\omega_2} +g^<_{\bm k_1,\omega_1} \phi(\bm q,\Omega) G^a_{\bm k_1+\bm q,\bm k_2,\omega_1+\Omega,\omega_2} \right]

電場ポテンシャルの取り込み次数で展開する

(9.4J) の右辺に (9.4I) と (9.4J) を代入すると、

(9.4K)

&G^<_{\bm k_1,\bm k_2,\omega_1,\omega_2}= 2\pi\delta(\omega_1-\omega_2)\delta_{\bm k_1,\bm k_2}g^<_{\bm k_1,\omega_1}\\ &+e\sum_{\bm q}\int d\frac{\Omega}{2\pi}\Bigg[ g^r_{\bm k_1,\omega_1} \phi(\bm q,\Omega) \Big[2\pi\delta(\omega_1+\Omega-\omega_2)\delta_{\bm k_1+\bm q,\bm k_2}g^<_{\bm k_2,\omega_2}\Big] +g^<_{\bm k_1,\omega_1} \phi(\bm q,\Omega) \Big[2\pi\delta(\omega_1+\Omega-\omega_2)\delta_{\bm k_1+\bm q,\bm k_2}g^a_{\bm k_2,\omega_2}\Big] \Bigg] \\&= 2\pi\delta(\omega_1-\omega_2)\delta_{\bm k_1,\bm k_2}g^<_{\bm k_1,\omega_1} +e \phi(\bm k_2-\bm k_1,\omega_2-\omega_1) \Big[ g^r_{\bm k_1,\omega_1} g^<_{\bm k_2,\omega_2} +g^<_{\bm k_1,\omega_1} g^a_{\bm k_2,\omega_2} \Big]+\cdots \\&\equiv 2\pi\delta(\omega_1-\omega_2)\delta_{\bm k_1,\bm k_2}g^<_{\bm k_1,\omega_1} +e \phi(\bm k_2-\bm k_1,\omega_2-\omega_1) \Big[ g_{\bm k_1,\omega_1} g_{\bm k_2,\omega_2} \Big]^<+\cdots

したがって、

(9.4)

&G^<_{\bm k-\frac{\bm q}{2},\bm k+\frac{\bm q}{2},\omega-\frac{\Omega}{2},\omega+\frac{\Omega}{2}}= 2\pi\delta(\Omega)\delta_{\bm q,\bm 0}g^<_{\bm k-\frac{\bm q}{2},\omega-\frac{\Omega}{2}} +e \phi(\bm q,\Omega) \Big[ g_{\bm k-\frac{\bm q}{2},\omega-\frac{\Omega}{2}} g_{\bm k+\frac{\bm q}{2},\omega+\frac{\Omega}{2}} \Big]^<+\cdots

右辺第一項にはδがあるので、教科書では Green 関数の添え字を \bm k\omega としている。

電荷密度を \phi の効果の取り込み次数に分割して \phi の1次の項を \rho_\phi^{(0)} と書く。

(9.5A)

\rho=\rho_i+\rho_\phi^{(0)}+\cdots

\rho_i は不純物散乱のみを考えた電荷密度で、 (9.4) の第一項 2\pi\delta(\Omega)\delta_{\bm q,\bm 0}g^<_{\bm k-\frac{\bm q}{2},\omega-\frac{\Omega}{2}} の項に相当する部分。

\phi の効果の最低次は、

\rho_\phi^{(0)} \propto e \phi(\bm q,\Omega) \Big[ g_{\bm k-\frac{\bm q}{2},\omega-\frac{\Omega}{2}} g_{\bm k+\frac{\bm q}{2},\omega+\frac{\Omega}{2}} \Big]^<

で、これって1次なのにどうして (0) がついてるの??? と思うわけだけど・・・

→ このゼロは \phi に関するゼロ次ではなく、 後に見る v_\phi をまたぐような不純物散乱の効果に対する取り込み次数らしい

\phi の高次の効果は始めから考えていない(線形応答の範囲で考える)

(9.5)

&\rho_\phi^{(0)}(\bm r,t)=-ie\frac{\textcolor{red}{\hbar}}{V} \int \frac{d\omega}{2\pi} \int \frac{d\Omega}{2\pi} \sum_{\bm k,\bm q} e^{-i\bm q\cdot \bm r} e^{i\Omega t} \ e \phi(\bm q,\Omega) \Big[ g_{\bm k-\frac{\bm q}{2},\omega-\frac{\Omega}{2}} g_{\bm k+\frac{\bm q}{2},\omega+\frac{\Omega}{2}} \Big]^< \\&= -i\frac{\textcolor{red}{e^2\hbar}}{V} \int \frac{d\omega}{2\pi} \int \frac{d\Omega}{2\pi} \sum_{\bm k,\bm q} e^{-i\bm q\cdot \bm r} e^{i\Omega t} \ \phi(\bm q,\Omega) \Big[ g_{\bm k-\frac{\bm q}{2},\omega-\frac{\Omega}{2}} g_{\bm k+\frac{\bm q}{2},\omega+\frac{\Omega}{2}} \Big]^< \\&= -i\frac{\textcolor{red}{e^2\hbar}}{a^3} \int \frac{d\omega}{2\pi} \int \frac{d\Omega}{2\pi} \frac{1}{N} \sum_{\bm k,\bm q} e^{-i\bm q\cdot \bm r} e^{i\Omega t} \ \phi(\bm q,\Omega) \Big[ g_{\bm k-\frac{\bm q}{2},\omega-\frac{\Omega}{2}} g_{\bm k+\frac{\bm q}{2},\omega+\frac{\Omega}{2}} \Big]^<

図 9.1 の見方は徐々に分かってきたけれど、
上と下のどちらが \textstyle \omega-\frac{\Omega}{2} なのか、
左と右のどちらが v_\phi なのかが分からないので、
セミナーの時に教えてもらう予定。

電場展開の最低次項を評価する

(9.5) の括弧の中身を (9.4K) の括弧の中身と (8.153) を使って展開してみる。

(9.6)

&\Big[ g_{\bm k-\frac{\bm q}{2},\omega-\frac{\Omega}{2}} g_{\bm k+\frac{\bm q}{2},\omega+\frac{\Omega}{2}} \Big]^< = \Big[ g^r_{\bm k-\frac{\bm q}{2},\omega-\frac{\Omega}{2}} g^<_{\bm k+\frac{\bm q}{2},\omega+\frac{\Omega}{2}} +g^<_{\bm k-\frac{\bm q}{2},\omega-\frac{\Omega}{2}} g^a_{\bm k+\frac{\bm q}{2},\omega+\frac{\Omega}{2}} \Big] \\ &= \Big[ g^r_{\bm k-\frac{\bm q}{2},\omega-\frac{\Omega}{2}} f(\omega+{\textstyle \frac{\Omega}{2}}) \Big( g^a_{\bm k+\frac{\bm q}{2},\omega+\frac{\Omega}{2}} - g^r_{\bm k+\frac{\bm q}{2},\omega+\frac{\Omega}{2}} \Big) + f(\omega-{\textstyle \frac{\Omega}{2}}) \Big( g^a_{\bm k-\frac{\bm q}{2},\omega-\frac{\Omega}{2}} - g^r_{\bm k-\frac{\bm q}{2},\omega-\frac{\Omega}{2}} \Big) g^a_{\bm k+\frac{\bm q}{2},\omega+\frac{\Omega}{2}} \Big] \\ &= \Big[ \Big( f(\omega+{\textstyle \frac{\Omega}{2}}) - f(\omega-{\textstyle \frac{\Omega}{2}}) \Big) g^r_{\bm k-\frac{\bm q}{2},\omega-\frac{\Omega}{2}} g^a_{\bm k+\frac{\bm q}{2},\omega+\frac{\Omega}{2}} - f(\omega+{\textstyle \frac{\Omega}{2}}) g^r_{\bm k-\frac{\bm q}{2},\omega-\frac{\Omega}{2}} g^r_{\bm k+\frac{\bm q}{2},\omega+\frac{\Omega}{2}} + f(\omega-{\textstyle \frac{\Omega}{2}}) g^a_{\bm k-\frac{\bm q}{2},\omega-\frac{\Omega}{2}} g^a_{\bm k+\frac{\bm q}{2},\omega+\frac{\Omega}{2}} \Big]

簡略化して表示すると、

(9.6A)

&\Big[g_-g_+\Big]^< = \Big[\Big( f(+) - f(-) \Big)g^r_-g^a_+ - f(+) g^r_-g^r_+ + f(-) g^a_-g^a_+ \Big]

f(ω) に含まれる Ω を展開する

\Omega \omega よりも十分小さいとして展開する。

f(\omega\pm{\textstyle \frac{\Omega}{2}})=f(\omega)\pm{\textstyle \frac{\Omega}{2}}f'(\omega)

(9.6B), (9.7)

&\Big[g_-g_+\Big]^< = \Big[\Big( f + \textstyle{\frac{\Omega}{2}}f' - f + \textstyle{\frac{\Omega}{2}}f' \Big)g^r_-g^a_+ - (f+\textstyle{\frac{\Omega}{2}}f') g^r_-g^r_+ + (f-\textstyle{\frac{\Omega}{2}}f') g^a_-g^a_+ \Big] \\&= \Big[\Omega f'(\omega) \Big( g^r_-g^a_+ - {\textstyle\frac{1}{2}} (g^r_-g^r_+ + g^a_-g^a_+)\Big) +f(\omega)(g^a_-g^a_+ - g^r_-g^r_+) \Big]

したがって、

&\rho_\phi^{(0)}(\bm r,t)= -i\frac{\textcolor{red}{e^2\hbar}}{a^3} \int \frac{d\Omega}{2\pi} \sum_{\bm q} e^{-i\bm q\cdot \bm r} e^{i\Omega t}\ \phi(\bm q,\Omega) \times \\ &\hspace{1cm}\frac{1}{N} \sum_{\bm k}\int \frac{d\omega}{2\pi} \Big[\Omega f'(\omega) \Big( g^r_-g^a_+ - {\textstyle\frac{1}{2}} (g^r_-g^r_+ + g^a_-g^a_+)\Big) +f(\omega)(g^a_-g^a_+ - g^r_-g^r_+) \Big]

g の添え字の q, Ω を展開する(最後には無視する)

これを具体的に評価するために、Green 関数を \bm q,\Omega の1次までで展開してみた。

この部分、教科書ではまるっきり飛ばされているのかと思いきや、 (9.33)〜(9.36) あたりで一応解説されていて、実は q については2次の項まで残さないといけないらしい。 2次の項まで残したやり方はどうせあとでやるので、ここでは1次まででやってみて、 どうして2次まで必要かを確認する。

g^r_\pm=\frac{1}{\hbar(\omega\pm\textstyle\frac{\Omega}{2})-\varepsilon_{\bm k\pm\textstyle\frac{\bm q}{2}}+\textstyle\frac{i\hbar}{2\tau}}

g^a_\pm=\frac{1}{\hbar(\omega\pm\textstyle\frac{\Omega}{2})-\varepsilon_{\bm k\pm\textstyle\frac{\bm q}{2}}-\textstyle\frac{i\hbar}{2\tau}}

g^\alpha_{\bm k\pm\frac{\bm q}{2},\omega\pm\frac{\Omega}{2}} \sim g^\alpha_{\bm k,\omega} \pm \frac{\bm q}{2}\cdot\nabla_{\bm k}g^\alpha_{\bm k,\omega} \pm \frac{\Omega}{2}\cdot\frac{\PD}{\PD \omega}g^\alpha_{\bm k,\omega}

\nabla_{\bm k}g^\alpha_{\bm k,\omega} = \nabla_{\bm k}\left(\frac{1}{\hbar\omega-\varepsilon_{\bm k}\pm\frac{i\hbar}{2\tau}}\right) = \frac{\nabla_{\bm k}\varepsilon_{\bm k}}{(\hbar\omega-\varepsilon_{\bm k}\pm\frac{i\hbar}{2\tau})^2} = \frac{\nabla_{\bm k}\left(\frac{\hbar^2}{2m}k^2-\varepsilon_F\right)}{(\hbar\omega-\varepsilon_{\bm k}\pm\frac{i\hbar}{2\tau})^2} = \frac{\frac{\hbar^2}{m}\bm k}{(\hbar\omega-\varepsilon_{\bm k}\pm\frac{i\hbar}{2\tau})^2} = \frac{\hbar^2\bm k}{m} g^\alpha{}_{\bm k,\omega}^2

\frac{\PD}{\PD \omega}g^\alpha_{\bm k,\omega} = \frac{\PD}{\PD \omega}\left(\frac{1}{\hbar\omega-\varepsilon_{\bm k}\pm\frac{i\hbar}{2\tau}}\right) = \frac{-\hbar}{(\hbar\omega-\varepsilon_{\bm k}\pm\frac{i\hbar}{2\tau})^2} = -\hbar g^\alpha_{\bm k,\omega}{}^2

したがって、

g^\alpha_{\bm k\pm\frac{\bm q}{2},\omega\pm\frac{\Omega}{2}} \sim g^\alpha_{\bm k,\omega} \pm \frac{\bm q}{2}\cdot\frac{\hbar^2\bm k}{m} g^\alpha_{\bm k,\omega}{}^2 \mp \frac{\Omega}{2}\cdot\hbar g^\alpha_{\bm k,\omega} {}^2 = g^\alpha_{\bm k,\omega} \pm \left[ \frac{\hbar^2\bm k\cdot\bm q}{2m} - \frac{\hbar\Omega}{2} \right] g^\alpha_{\bm k,\omega}{}^2 \equiv g^\alpha_{\bm k,\omega} \pm \Delta_{\bm q,\Omega} \, g^\alpha_{\bm k,\omega}{}^2

すなわち、

\Delta_{\bm q,\Omega} = \frac{\hbar^2\bm k\cdot\bm q}{2m} - \frac{\hbar\Omega}{2}

これを用いれば、

&\frac{1}{N} \sum_{\bm k} \int \frac{d\omega}{2\pi} \Omega f'(\omega) g^r_-g^a_+ = \frac{1}{N} \sum_{\bm k} \int \frac{d\omega}{2\pi} \Omega f'(\omega) \Big(g^r_{\bm k,\omega}-\Delta_{\bm q,\Omega}\,g^r_{\bm k,\omega}{}^2+\dots\Big) \Big(g^a_{\bm k,\omega}+\Delta_{\bm q,\Omega}\,g^a_{\bm k,\omega}{}^2+\dots\Big) \\&= \Omega \int \frac{d\omega}{2\pi} f'(\omega) \frac{1}{N} \sum_{\bm k} \Big[ g^r_{\bm k,\omega}g^a_{\bm k,\omega} +\Delta_{\bm q,\Omega}\,g^r_{\bm k,\omega}g^a_{\bm k,\omega}{}^2 -\Delta_{\bm q,\Omega}\,g^a_{\bm k,\omega}g^r_{\bm k,\omega}{}^2 -\Delta_{\bm q,\Omega}{}^2\,g^r_{\bm k,\omega}{}^2g^a_{\bm k,\omega}{}^2+\dots \Big]

Σg を評価する

そこで 2 \le n + m として、

\frac{1}{N} \sum_{\bm k} g^r_{\bm k,\omega}{}^m g^a_{\bm k,\omega}{}^n = \int_{-\varepsilon_F}^\infty d\varepsilon \, \nu(\varepsilon) \frac{1}{(\hbar\omega-\varepsilon+i\delta_\varepsilon)^m} \frac{1}{(\hbar\omega-\varepsilon-i\delta_\varepsilon)^n}

を評価したい。( \delta_\varepsilon=\frac{\hbar}{2\tau} )

(9.18) と同様の手順を考えると、被積分関数は \hbar\omega+i\delta_\varepsilon m 次の \hbar\omega-i\delta_\varepsilon n 次の極を持つから、 被積分関数を h(z) とすれば、

\frac{1}{N} \sum_{\bm k} g^r_{\bm k,\omega}{}^m g^a_{\bm k,\omega}{}^n = \pi i\Big(\Res_{z=\hbar\omega+i\delta_\varepsilon}h(z)+\Res_{z=\hbar\omega-i\delta_\varepsilon}h(z)\Big)

ただし( n m は適宜読み替えて)、

&\Res_{z=\hbar\omega\pm i\delta_\varepsilon}h(z) =\frac{1}{(n-1)!}\frac{d^{n-1}}{dz^{n-1}}\Big((z-\hbar\omega\pm i\delta_\varepsilon)^nh(z)\Big) \bigg|_{z=\hbar\omega\pm i\delta_\varepsilon} \\&= \frac{1}{(n-1)!}\frac{d^{n-1}}{dz^{n-1}}\frac{(-1)^n\nu(z)}{(\hbar\omega-z\mp i\delta_\varepsilon)^m} \Bigg|_{z=\hbar\omega\pm i\delta_\varepsilon} \\&= \frac{(-1)^n}{(n-1)!} \sum_{i=0}^{n-1} {}_{n-1}\mathrm{C}_i \frac{d^{i}}{dz^{i}}\Bigg(\frac{1}{(\hbar\omega-z\mp i\delta_\varepsilon)^m}\Bigg) \frac{d^{n-1-i}\nu(z)}{dz^{n-1-i}} \Bigg|_{z=\hbar\omega\pm i\delta_\varepsilon} \\&= \frac{(-1)^n}{(n-1)!} \sum_{i=0}^{n-1}{}_{n-1}\mathrm{C}_i {\ }_{m+i}\mathrm{C}_{i}\frac{1}{(\pm 2i\delta_\varepsilon)^{m+i}} \frac{d^{n-1-i}\nu(z)}{dz^{n-1-i}} \Bigg|_{z=\hbar\omega\pm i\delta_\varepsilon}

である。 \nu の微分は、

\nu'(z)=\frac{m^3/2a^3}{\sqrt{2}\pi^2\hbar^3}\Big([z+\varepsilon_F]^{1/2}\Big)' =\frac{m^3/2a^3}{\sqrt{2}\pi^2\hbar^3}\Big(\frac{1}{2}[z+\varepsilon_F]^{-1/2}\Big) = \frac{1}{2}\frac{1}{z+\varepsilon_F}\nu(z)

\nu''(z) =\frac{m^3/2a^3}{\sqrt{2}\pi^2\hbar^3}\Big(\frac{1}{2}\Big(-\frac{1}{2}\Big)[z+\varepsilon_F]^{-3/2}\Big) = \frac{1}{2}\Big(-\frac{1}{2}\Big)\frac{1}{(z+\varepsilon_F)^2}\nu(z)

\nu'''(z) =\frac{m^3/2a^3}{\sqrt{2}\pi^2\hbar^3}\Big(\frac{1}{2}\Big(-\frac{1}{2}\Big)\Big(-\frac{3}{2}\Big)[z+\varepsilon_F]^{-5/2}\Big) = \frac{1}{2}\Big(-\frac{1}{2}\Big)\Big(-\frac{3}{2}\Big)\frac{1}{(z+\varepsilon_F)^3}\nu(z)

\frac{d^n\nu(z)}{dz^n} = (-1)^{n-1}\frac{(2n-3)!!}{2^n}\frac{\nu(z)}{(z+\varepsilon_F)^n}

すなわち、

&\nu'(z)|_{z=\hbar\omega\pm i\delta_\varepsilon}=\frac{1}{2}\frac{\nu(\hbar\omega\pm i\delta_\varepsilon)}{\hbar\omega+\varepsilon_F\pm i\delta_\varepsilon} \sim \pm\frac{\nu(0)}{2\varepsilon_F}

\frac{d^n\nu(z)}{dz^n}\Big|_{z=\hbar\omega\pm i\delta_\varepsilon} = (-1)^{n-1}\frac{(2n-3)!!}{2^n}\frac{\nu(\hbar\omega\pm i\delta_\varepsilon)}{(\hbar\omega+\varepsilon_F\pm i\delta_\varepsilon)^n} \sim \pm(-1)^{n-1}\frac{(2n-3)!!}{(2\varepsilon_F)^n}\nu(0)

である。先頭に出てくる複号は、 \nu(z) のカットが実数軸に沿って走るため、 \hbar\omega\pm i\delta_\varepsilon が上半面にあるか下半面にあるかで符号が異なるためである。

f'(ω) に掛かる第1項の主項 : (grga)

これを用いると(後で出てくる \hbar\omega=0 を先取りして使う)、

&\frac{1}{N} \sum_{\bm k} g^r g^a = \pi i \left[ - \frac{\nu(\hbar\omega+i\delta_\varepsilon)}{-2i\delta_\varepsilon} - \frac{\nu(\hbar\omega-i\delta_\varepsilon)}{ 2i\delta_\varepsilon} \right] = \frac{\pi}{2\delta_\varepsilon}\Big[ \nu(\hbar\omega+i\delta_\varepsilon) - \nu(\hbar\omega-i\delta_\varepsilon) \Big] \\ &= \frac{\pi}{2\delta_\varepsilon}\Big[ \nu(0) + \nu(0) \Big] = \frac{\pi}{\delta_\varepsilon} \nu(0) = \frac{2\pi\tau}{\hbar} \nu(0)

低温では (9.12) に示されるように f'(\omega)=-\delta(\omega) と見なせるため、

(9.8-1)

\Omega \int \frac{d\omega}{2\pi} f'(\omega) \frac{1}{N}\sum_{\bm k} g^r_{\bm k,\omega}g^a_{\bm k,\omega} = \Omega \int \frac{d\omega}{2\pi} f'(\omega) \frac{2\pi\tau}{\hbar} \nu(0) = -\Omega \int d\omega \delta(\omega) \frac{\tau}{\hbar} \nu(0) = - \Omega\tau \frac{\nu(0)}{\hbar}

f'(ω) に掛かる第2項の主項 : (gaga+grgr)

また、こちらは (8.137) でも評価済みであるが、

&\frac{1}{N} \sum_{\bm k} g^a_{\bm k,\omega} g^a_{\bm k,\omega} = \pi i \nu'(\hbar\omega-i\delta_\varepsilon) = i\frac{\pi}{2} \frac{\nu(\hbar\omega-i\delta_\varepsilon)}{\hbar\omega+\varepsilon_F-i\delta_\varepsilon}

より、

(9.8-2)

&\Omega \int \frac{d\omega}{2\pi} f'(\omega) \frac{1}{N}\sum_{\bm k} g^a_{\bm k,\omega}g^a_{\bm k,\omega} = i\Omega \int \frac{d\omega}{2\pi} f'(\omega) \frac{\pi}{2} \frac{\nu(\hbar\omega-i\delta_\varepsilon)}{\hbar\omega+\varepsilon_F-i\delta_\varepsilon} = -i\Omega\frac{\pi}{2} \frac{\nu(-i\delta_\varepsilon)}{\varepsilon_F-i\delta_\varepsilon}\\ &\sim i\Omega\frac{\pi}{2} \frac{\nu(0)}{\varepsilon_F} = \textcolor{red}{+}\frac{\pi i}{2}\Omega \frac{2\tau}{\hbar} \nu(0)\left(\frac{\hbar}{2\tau\varepsilon_F}\right)

ただしこれらは \Delta_{\bm q,\Omega} の影響を除いた評価になっている。

f'(ω) に掛かる項の q, Ω 成分

\Delta_{\bm q,\Omega} = \frac{\hbar^2\bm k\cdot\bm q}{2m} - \frac{\hbar\Omega}{2}

の掛かっている部分の和を取ろうとすると、 \Delta_{\bm q,\Omega} \bm k を含むために計算がしづらい。

以下で見る f(\omega) の項については g^r g^a だけしか掛かっていなかったのでうまく k に関する部分積分が使えるけれど、 g^r と  g^a がまぜこぜに掛かっていると、同じ方法では評価できないから。

とはいえ、考え方としては f'(\omega) の項はそもそも \textstyle f(\omega\pm\frac{\Omega}{2}) \Omega で展開した副次項であり、そのさらに副次項である \Delta_{\bm q,\Omega} の項をそこまで厳密に評価しなくても無視して良いのかもしれない。

→ この点について、なぜかずいぶん後の (9.33)〜(9.37) で詳しく見ることになる

そう言う意味で、 \textstyle f(\omega\pm\frac{\Omega}{2}) の主項 f(\omega) に掛かる副次項は以下でしっかり評価することになる。

f(ω) に掛かる項

(9.7) の f(\omega) の項について見てみる。

&\hspace{1cm}\frac{1}{N} \sum_{\bm k}\int \frac{d\omega}{2\pi} f(\omega)(g^a_-g^a_+ - g^r_-g^r_+)

の第1項は、

(9.10)

&\frac{1}{N} \sum_{\bm k}\int \frac{d\omega}{2\pi} f(\omega)g^a_-g^a_+ \\&\sim \frac{1}{N} \sum_{\bm k}\int \frac{d\omega}{2\pi} f(\omega) \big(g^\alpha_{\bm k,\omega} + \Delta_{\bm q,\Omega} \, g^\alpha_{\bm k,\omega}{}^2\big) \big(g^\alpha_{\bm k,\omega} - \Delta_{\bm q,\Omega} \, g^\alpha_{\bm k,\omega}{}^2\big) \\&= \frac{1}{N} \sum_{\bm k}\int \frac{d\omega}{2\pi} f(\omega) \big(\textcolor{red}{g^\alpha_{\bm k,\omega}{}^2 - \Delta_{\bm q,\Omega}{}^2 \, g^\alpha_{\bm k,\omega}{}^4}\big)

じゃないのだろうか? g^3 の項が出る理由が分からない???

→ (9.35) あたりで詳しくやるためここでは詳細を省くが、 q については2乗の項まで考慮する必要がある。 もともと q の1次ではなく2次の項まで残しておくと、 その2次の項と0次の項のかけ算で、 q の2次を係数に持つ g^3 の項が出る。 1次の項までで評価した上式が、最終的に q の2次の効果しか生まないため、 もともと q の2次だった項が1次と同程度の寄与を生むことになる。

&\frac{1}{N} \sum_{\bm k}\int \frac{d\omega}{2\pi} f(\omega)g^a_-g^a_+ \\&\sim \frac{1}{N} \sum_{\bm k}\int \frac{d\omega}{2\pi} f(\omega) \big(g^\alpha_{\bm k,\omega}{}^2 + \frac{\hbar^2q^2}{4m}g^\alpha_{\bm k,\omega}{}^3 - \Delta_{\bm q,\Omega}{}^2 \, g^\alpha_{\bm k,\omega}{}^4\big)

ここで、

&\frac{\PD}{\PD \omega}g^\alpha_{\bm k,\omega} = \frac{\PD}{\PD \omega}\frac{1}{\hbar\omega-\varepsilon\pm i\delta_\varepsilon} \\&= \frac{-\hbar}{(\hbar\omega-\varepsilon\pm i\delta_\varepsilon)^2} = -\hbar g^\alpha_{\bm k,\omega}{}^2

そこで、

g^\alpha_{\bm k,\omega}{}^2=-\frac{1}{\textcolor{red}{\hbar}}\frac{\PD}{\PD \omega}g^\alpha_{\bm k,\omega}

を使って書き換えると、

(9.11)

&\frac{1}{N} \sum_{\bm k}\int \frac{d\omega}{2\pi} f(\omega)g^a_-g^a_+ \\&= \frac{1}{\textcolor{red}{\hbar}N} \sum_{\bm k}\int \frac{d\omega}{2\pi} f(\omega) \Big(-\frac{\PD g^\alpha_{\bm k,\omega}}{\PD \omega} + \frac{\hbar^2q^2}{4m} \, g^\alpha_{\bm k,\omega}\frac{\PD g^\alpha_{\bm k,\omega}}{\PD \omega} + \Delta_{\bm q,\Omega}{}^2 \, g^\alpha_{\bm k,\omega}{}^2\frac{\PD g^\alpha_{\bm k,\omega}}{\PD \omega}\Big) \\&= \frac{1}{\textcolor{red}{\hbar}N} \sum_{\bm k} \Bigg( -\Big[f(\omega) \Big(g^\alpha_{\bm k,\omega} + \frac{1}{2} \frac{\hbar^2q^2}{4m} \, g^\alpha_{\bm k,\omega}{}^2 - \frac{1}{3} \Delta_{\bm q,\Omega}{}^2 \, g^\alpha_{\bm k,\omega}{}^3\Big) \Big]_{-\infty}^\infty+ \int \frac{d\omega}{2\pi} f'(\omega) \big(g^\alpha_{\bm k,\omega} + \frac{1}{2} \frac{\hbar^2q^2}{4m} \, g^\alpha_{\bm k,\omega}{}^2 - \frac{1}{3} \Delta_{\bm q,\Omega}{}^2 \, g^\alpha_{\bm k,\omega}{}^3\big) \Bigg) \\&= \frac{1}{\textcolor{red}{\hbar}N} \sum_{\bm k} \int \frac{d\omega}{2\pi} f'(\omega) \Big(g^\alpha_{\bm k,\omega} + \frac{\hbar^2q^2}{8m} g^\alpha_{\bm k,\omega}{}^2 - \frac{1}{3} \Delta_{\bm q,\Omega}{}^2 \, g^\alpha_{\bm k,\omega}{}^3\Big)

\omega\rightarrow\pm\infty g\rightarrow 0 を用いて部分積分を行った。

f(\omega) は低温でステップ関数となるため、その微分はδ関数と見なせる。

(9.12), (9.13)

f'(\omega)=-\delta(\omega)

そこで、

(9.14)

&\frac{1}{N} \sum_{\bm k}\int \frac{d\omega}{2\pi} f(\omega)g^a_-g^a_+ \\&= -\frac{1}{\textcolor{red}{\hbar}N} \sum_{\bm k} \int \frac{d\omega}{2\pi} \delta(\omega) \Big(g^\alpha_{\bm k,\omega} +\frac{\hbar^2q^2}{8m} g^\alpha_{\bm k,\omega}{}^2 - \frac{1}{3} \Delta_{\bm q,\Omega}{}^2 \, g^\alpha_{\bm k,\omega}{}^3\Big) \\&= -\frac{1}{2\pi\textcolor{red}{\hbar} N} \sum_{\bm k} \Big(g^\alpha_{\bm k,0} +\frac{\hbar^2q^2}{8m} g^\alpha_{\bm k,\omega}{}^2 - \frac{1}{3} \Delta_{\bm q,\Omega}{}^2 \, g^\alpha_{\bm k,0}{}^3\Big) \\&\equiv -\frac{1}{2\pi\textcolor{red}{\hbar} N} \sum_{\bm k} \Big(g^\alpha_{\bm k} +\frac{\hbar^2q^2}{8m} g^\alpha_{\bm k,\omega}{}^2 - \frac{1}{3} \Delta_{\bm q,\Omega}{}^2 \, g^\alpha_{\bm k}{}^3\Big)

となる。 g_{\bm k}^\alpha=g_{\bm k,\omega=0}^\alpha である。

ここへ来てようやく、 \sum g のような計算の中で暗黙のうちに \hbar\omega=0 が仮定されていた理由が分かった。

第1項、第2項は簡単に評価できるので、以下では第3項について考える。

f(ω) にかかる q に依存する項

まずは \Delta_{\bm q,\Omega} の中の \bm q 依存する項のみ考える。

&\sum_{\bm k}\Big(\frac{\hbar^2}{2m}\Big)^2(\bm q\cdot\bm k)^2 g_{\bm k}^\alpha{}^3 = \Big(\frac{\hbar^2}{4m}\Big)\sum_{i=x,y,z} \sum_{j=x,y,z} q_i q_j \sum_{\bm k} k_i k_j g_{\bm k}^\alpha \Big(\frac{\hbar^2}{m}\Big) g_{\bm k}^\alpha{}^2 \\&= \Big(\frac{\hbar^2}{4m}\Big)\sum_{i=x,y,z} \sum_{j=x,y,z} q_i q_j \sum_{\bm k} k_j g_{\bm k}^\alpha \frac{\PD g_{\bm k}^\alpha}{\PD k_i}

この &\sum_{\bm k} k_j g_{\bm k}^\alpha \frac{\PD g_{\bm k}^\alpha}{\PD k_i} について、

i\ne j の時には、

&\sum_{\bm k} k_j g_{\bm k}^\alpha \frac{\PD g_{\bm k}^\alpha}{\PD k_i} = \sum_{k_k}\sum_{k_j}\sum_{k_i} k_j g_{\bm k}^\alpha \frac{\PD g_{\bm k}^\alpha}{\PD k_i} = \sum_{k_k}\sum_{k_i}\left(\sum_{k_j} k_j\right) g_{\bm k}^\alpha \frac{\PD g_{\bm k}^\alpha}{\PD k_i} = \sum_{\bm k_k}\sum_{\bm k_i} 0\, g_{\bm k}^\alpha \frac{\PD g_{\bm k}^\alpha}{\PD k_i} = 0

i = j の時には、

&\sum_{\bm k} k_i g_{\bm k}^\alpha \frac{\PD g_{\bm k}^\alpha}{\PD k_i} = \sum_{\bm k_k}\sum_{\bm k_j}\sum_{\bm k_i} k_i \left(\frac{1}{2}g_{\bm k}^\alpha{}^2\right)' = \sum_{\bm k_k}\sum_{\bm k_j} \left[ k_i \left(\frac{1}{2}g_{\bm k}^\alpha{}^2\right)' \right] -\sum_{\bm k_k}\sum_{\bm k_j} \sum_{\bm k_i} \frac{1}{2}g_{\bm k}^\alpha{}^2\\ &=-\sum_{\bm k}\frac{1}{2}g_{\bm k}^\alpha{}^2\\

したがって、

&\sum_{\bm k} k_i g_{\bm k}^\alpha \frac{\PD g_{\bm k}^\alpha}{\PD k_i} =-\sum_{\bm k}\frac{1}{2}\delta_{ij}g_{\bm k}^\alpha{}^2\\

これを使って、

(9.16)

&\sum_{\bm k}\Big(\frac{\hbar^2}{2m}\Big)^2(\bm q\cdot\bm k)^2 g_{\bm k}^\alpha{}^3 = -\Big(\frac{\hbar^2}{4m}\Big)\sum_{i=x,y,z} \sum_{j=x,y,z} q_i q_j \sum_{\bm k}\frac{1}{2}\delta_{ij}g_{\bm k}^\alpha{}^2 \\&= -\Big(\frac{\hbar^2}{4m}\Big)\sum_{i=x,y,z} q_i^2 \sum_{\bm k}\frac{1}{2}g_{\bm k}^\alpha{}^2 \\&= -\Big(\frac{\hbar^2q^2}{8m}\Big)\sum_{\bm k} g_{\bm k}^\alpha{}^2

すなわち、

(9.17)

\frac{1}{N} \sum_{\bm k}\int \frac{d\omega}{2\pi} f(\omega)g^a_{\bm k-\frac{\bm q}{2},\omega}g^a_{\bm k+\frac{\bm q}{2},\omega} &= -\frac{1}{2\pi\textcolor{red}{\hbar} N} \sum_{\bm k} \big(g^\alpha_{\bm k} + \Big(\frac{\hbar^2q^2}{8m}\Big) \, g^\alpha_{\bm k}{}^2 + \frac{1}{3} \Big(\frac{\hbar^2q^2}{8m}\Big) \, g^\alpha_{\bm k}{}^2\big) \\&= -\frac{1}{2\pi\textcolor{red}{\hbar} N} \sum_{\bm k} \big(g^\alpha_{\bm k} + \Big(\frac{\hbar^2q^2}{6m}\Big) \, g^\alpha_{\bm k}{}^2\big)

(9.18)〜(9.22) はすでに何回も使っている → δ関数の性質

ただ、教科書のの計算は途中まで g^r の物になっているので注意。

(9.23)

&\frac{1}{N} \sum_{\bm k}\int \frac{d\omega}{2\pi} f(\omega)g^a_{\bm k-\frac{\bm q}{2},\omega}g^a_{\bm k+\frac{\bm q}{2},\omega} = -\frac{1}{2\pi\hbar}\left(+\pi i\nu(0)\right) \left[1 + \frac{1}{2\varepsilon_F} \Big(\frac{\hbar^2q^2}{6m}\Big) \right] = -\frac{i\nu(0)}{2\hbar} \left[1 - \frac{1}{6} \Big(\frac{q}{k_F}\Big)^2 \right]

ここで、 \varepsilon_F=\frac{\hbar^2k_F^2}{2m} を用いた。

同様にして、

&\frac{1}{N} \sum_{\bm k}\int \frac{d\omega}{2\pi} f(\omega)g^r_{\bm k-\frac{\bm q}{2},\omega}g^r_{\bm k+\frac{\bm q}{2},\omega} = \frac{i\nu(0)}{2\hbar} \left[1 - \frac{1}{6} \Big(\frac{q}{k_F}\Big)^2 \right]

より、

&\frac{1}{N} \sum_{\bm k}\int \frac{d\omega}{2\pi} f(\omega)\Big( g^a_{\bm k-\frac{\bm q}{2},\omega}g^a_{\bm k+\frac{\bm q}{2},\omega} -g^r_{\bm k-\frac{\bm q}{2},\omega}g^r_{\bm k+\frac{\bm q}{2},\omega}\Big) = -\frac{i\nu(0)}{\hbar} \left[1 - \frac{1}{6} \Big(\frac{q}{k_F}\Big)^2 \right] \sim -\frac{i\nu(0)}{\hbar}

第2項は第1項に比べて \Big(\frac{q}{k_F}\Big)^2 だけ小さく、無視できる。

f(ω) にかかる Ω2 に依存する項

\Omega/2 の項は、

&\frac{1}{2\pi\textcolor{red}{\hbar} N} \sum_{\bm k} \frac{1}{3} \Delta_{\bm 0,\Omega}{}^2 \, g^\alpha_{\bm k}{}^3 \\&= \frac{1}{2\pi\textcolor{red}{\hbar}} \frac{1}{3} \left(-\frac{\textcolor{red}{\hbar} \Omega}{2}\right)^2 \frac{1}{N}\sum_{\bm k} g^\alpha_{\bm k}{}^3 \\&= -\frac{1}{2\pi\textcolor{red}{\hbar}} \frac{\textcolor{red}{\hbar^2}\Omega^2}{12} \pi i\nu''(-i\delta_\varepsilon) \\&= \frac{i}{2\textcolor{red}{\hbar}} \frac{\textcolor{red}{\hbar^2}\Omega^2}{12} \frac{\nu(-i\delta_\varepsilon)}{4\varepsilon_F{}^2} \\&= -\frac{1}{48}\left( \frac{\hbar \Omega}{\varepsilon_F} \right)^2 \frac{i\nu(0)}{2\hbar}

第1項に比べて \left( \frac{\hbar \Omega}{\varepsilon_F} \right)^2 だけ小さいことが分かった。

f(ω) にかかる q2Ω に依存する項

以下では q^2 \Omega は同じくらいか、 むしろ \Omega の方が小さいような扱いを受けている。

(9.24) が q^2\Omega に比例する項を見ているのはそのためのようである。 (この項が q=0 のときゼロになることと、 \Delta_{\bm q} g が最終的に q^2 に比例する項となることから、外からかかる \Omega と共に、この項が q^2\Omega に比例することは明らかである)

したがって、

(9.25)

&\frac{1}{\textcolor{red}{N}}\sum_{\bm k}\int \frac{d\omega}{2\pi} f(\omega)(g^a_-g^a_+-g^r_-g^r_+) \\&= \frac{1}{\textcolor{red}{N}}\sum_{\bm k}\int \frac{d\omega}{2\pi} f(\omega)(g^a_{\bm k,\omega}{}^2-g^r_{\bm k,\omega}{}^2) \left[ 1 + O\Big(\frac{q}{k_F}\Big)^2 + O\left( \frac{\hbar \Omega}{\varepsilon_F} \right)^2 \right] \\&= -\frac{i\nu(0)}{\hbar} \left[ 1 + O\Big(\frac{q}{k_F}\Big)^2 + O\left( \frac{\hbar \Omega}{\varepsilon_F} \right)^2 \right]

と評価できる。

最終的には

(9.26)

&\rho_\phi^{(0)}(\bm r,t)= -i\frac{\textcolor{red}{e^2\hbar}}{a^3} \int \frac{d\Omega}{2\pi} \sum_{\bm q} e^{-i\bm q\cdot \bm r} e^{i\Omega t}\ \phi(\bm q,\Omega) \times \\ &\hspace{1cm}\frac{1}{N} \sum_{\bm k}\int \frac{d\omega}{2\pi} \Big[\Omega f'(\omega) \Big( g^r_-g^a_+ - {\textstyle\frac{1}{2}} (g^r_-g^r_+ + g^a_-g^a_+)\Big) +f(\omega)(g^a_-g^a_+ - g^r_-g^r_+) \Big] \\&\sim -i\frac{\textcolor{red}{e^2\hbar}}{a^3} \int \frac{d\Omega}{2\pi} \sum_{\bm q} e^{-i\bm q\cdot \bm r} e^{i\Omega t}\ \phi(\bm q,\Omega) \times \\ &\hspace{1cm}\frac{1}{N} \sum_{\bm k}\int \frac{d\omega}{2\pi} \Big[\Omega f'(\omega) g^r_{\bm k,\omega}g^a_{\bm k,\omega} +f(\omega)(g^a_{\bm k,\omega}{}^2 - g^r_{\bm k,\omega}{}^2) \Big] \\&= -i\frac{\textcolor{red}{e^2\hbar}}{a^3} \int \frac{d\Omega}{2\pi} \sum_{\bm q} e^{-i\bm q\cdot \bm r} e^{i\Omega t}\ \phi(\bm q,\Omega) \times \left[ -\Omega\tau\frac{\nu(0)}{\hbar} -\frac{i\nu(0)}{\hbar} \right] \\&= -\frac{\textcolor{red}{e^2}}{a^3} \nu(0) \int \frac{d\Omega}{2\pi} \sum_{\bm q} e^{-i\bm q\cdot \bm r} e^{i\Omega t}\ \phi(\bm q,\Omega) \times \left[ 1-i\Omega\tau \right]

と評価できる。

質問・コメント





Counter: 4245 (from 2010/06/03), today: 2, yesterday: 0